Finding convergence of integral $int_0^1 frac{x^n}{1+x}dx$












0














Test the convergence $$int_0^1 frac{x^n}{1+x}dx$$



I have used comparison test for improper integrals..by comparing with $1/(1+x)$...
so I found it convergent ..
But the solution set says that it is convergent if $n> -1$.










share|cite|improve this question





























    0














    Test the convergence $$int_0^1 frac{x^n}{1+x}dx$$



    I have used comparison test for improper integrals..by comparing with $1/(1+x)$...
    so I found it convergent ..
    But the solution set says that it is convergent if $n> -1$.










    share|cite|improve this question



























      0












      0








      0







      Test the convergence $$int_0^1 frac{x^n}{1+x}dx$$



      I have used comparison test for improper integrals..by comparing with $1/(1+x)$...
      so I found it convergent ..
      But the solution set says that it is convergent if $n> -1$.










      share|cite|improve this question















      Test the convergence $$int_0^1 frac{x^n}{1+x}dx$$



      I have used comparison test for improper integrals..by comparing with $1/(1+x)$...
      so I found it convergent ..
      But the solution set says that it is convergent if $n> -1$.







      calculus






      share|cite|improve this question















      share|cite|improve this question













      share|cite|improve this question




      share|cite|improve this question








      edited Nov 24 at 18:05









      gimusi

      1




      1










      asked Nov 24 at 17:58









      Kashmira

      463




      463






















          3 Answers
          3






          active

          oldest

          votes


















          0














          We have that for $nge 0$ the integral is a proper integral, then consider $n<0$ and by $m=-n>0$ we have



          $$int_0^1 frac{1}{x^m+x^{m+1}}dx$$



          and since as $xto 0^+$



          $$frac{1}{x^m+x^{m+1}} sim frac{1}{x^m}$$



          the integral converges for $m<1$ that is $n>-1$.



          As an alternative by $y=frac1x$ we have



          $$int_0^1 frac{x^n}{1+x}dx=int_1^infty frac{1}{y^{n+1}+y^{n+2}}dx$$



          and since as $xto infty$



          $$ frac{1}{y^{n+1}+y^{n+2}}sim frac{1}{y^{n+2}}$$



          the integral converges for $n+2>1$ that is $n>-1$.






          share|cite|improve this answer





























            0














            Let $a_n = displaystyle int_{0}^1 dfrac{x^n}{1+x}dx$. Since $dfrac{x^n}{1+x} = x^{n-1}left(1-dfrac{1}{1+x}right)=x^{n-1}-dfrac{x^{n-1}}{1+x}$, taking the integral $displaystyle int_{0}^1$ both sides give: $a_n = dfrac{1}{n}- a_{n-1}=dfrac{1}{n}-dfrac{1}{n-1}+a_{n-2}= dfrac{1}{n}-dfrac{1}{n-1}+dfrac{1}{n-2}-a_{n-3}=...=-ln 2+ 1-dfrac{1}{2}+dfrac{1}{3}-dfrac{1}{4}+dfrac{1}{5}-dfrac{1}{6}+cdots + dfrac{1}{n-2}-dfrac{1}{n-1}+dfrac{1}{n}$. $a_n$ converges when considered as a partial sum of an alternating harmonic series.






            share|cite|improve this answer































              0














              Since $x^n$ is continuous on $[0,1]$ for $nge 0,$ the integral converges for $nge 0.$



              For $n<0,$ $x^n$ blows up at $0.$ So we need to consider the integral over $[a,1]$ for small $a>0.$ Notice that for $xin (0,1],$



              $$frac{x^n}{2}le frac{x^n}{1+x} le x^n.$$



              It follows that the integral of interest converges iff $int_0^1 x^n,dx $ converges. Things are easy now, so I'll stop here. Ask questions if you like.






              share|cite|improve this answer





















                Your Answer





                StackExchange.ifUsing("editor", function () {
                return StackExchange.using("mathjaxEditing", function () {
                StackExchange.MarkdownEditor.creationCallbacks.add(function (editor, postfix) {
                StackExchange.mathjaxEditing.prepareWmdForMathJax(editor, postfix, [["$", "$"], ["\\(","\\)"]]);
                });
                });
                }, "mathjax-editing");

                StackExchange.ready(function() {
                var channelOptions = {
                tags: "".split(" "),
                id: "69"
                };
                initTagRenderer("".split(" "), "".split(" "), channelOptions);

                StackExchange.using("externalEditor", function() {
                // Have to fire editor after snippets, if snippets enabled
                if (StackExchange.settings.snippets.snippetsEnabled) {
                StackExchange.using("snippets", function() {
                createEditor();
                });
                }
                else {
                createEditor();
                }
                });

                function createEditor() {
                StackExchange.prepareEditor({
                heartbeatType: 'answer',
                autoActivateHeartbeat: false,
                convertImagesToLinks: true,
                noModals: true,
                showLowRepImageUploadWarning: true,
                reputationToPostImages: 10,
                bindNavPrevention: true,
                postfix: "",
                imageUploader: {
                brandingHtml: "Powered by u003ca class="icon-imgur-white" href="https://imgur.com/"u003eu003c/au003e",
                contentPolicyHtml: "User contributions licensed under u003ca href="https://creativecommons.org/licenses/by-sa/3.0/"u003ecc by-sa 3.0 with attribution requiredu003c/au003e u003ca href="https://stackoverflow.com/legal/content-policy"u003e(content policy)u003c/au003e",
                allowUrls: true
                },
                noCode: true, onDemand: true,
                discardSelector: ".discard-answer"
                ,immediatelyShowMarkdownHelp:true
                });


                }
                });














                draft saved

                draft discarded


















                StackExchange.ready(
                function () {
                StackExchange.openid.initPostLogin('.new-post-login', 'https%3a%2f%2fmath.stackexchange.com%2fquestions%2f3011862%2ffinding-convergence-of-integral-int-01-fracxn1xdx%23new-answer', 'question_page');
                }
                );

                Post as a guest















                Required, but never shown

























                3 Answers
                3






                active

                oldest

                votes








                3 Answers
                3






                active

                oldest

                votes









                active

                oldest

                votes






                active

                oldest

                votes









                0














                We have that for $nge 0$ the integral is a proper integral, then consider $n<0$ and by $m=-n>0$ we have



                $$int_0^1 frac{1}{x^m+x^{m+1}}dx$$



                and since as $xto 0^+$



                $$frac{1}{x^m+x^{m+1}} sim frac{1}{x^m}$$



                the integral converges for $m<1$ that is $n>-1$.



                As an alternative by $y=frac1x$ we have



                $$int_0^1 frac{x^n}{1+x}dx=int_1^infty frac{1}{y^{n+1}+y^{n+2}}dx$$



                and since as $xto infty$



                $$ frac{1}{y^{n+1}+y^{n+2}}sim frac{1}{y^{n+2}}$$



                the integral converges for $n+2>1$ that is $n>-1$.






                share|cite|improve this answer


























                  0














                  We have that for $nge 0$ the integral is a proper integral, then consider $n<0$ and by $m=-n>0$ we have



                  $$int_0^1 frac{1}{x^m+x^{m+1}}dx$$



                  and since as $xto 0^+$



                  $$frac{1}{x^m+x^{m+1}} sim frac{1}{x^m}$$



                  the integral converges for $m<1$ that is $n>-1$.



                  As an alternative by $y=frac1x$ we have



                  $$int_0^1 frac{x^n}{1+x}dx=int_1^infty frac{1}{y^{n+1}+y^{n+2}}dx$$



                  and since as $xto infty$



                  $$ frac{1}{y^{n+1}+y^{n+2}}sim frac{1}{y^{n+2}}$$



                  the integral converges for $n+2>1$ that is $n>-1$.






                  share|cite|improve this answer
























                    0












                    0








                    0






                    We have that for $nge 0$ the integral is a proper integral, then consider $n<0$ and by $m=-n>0$ we have



                    $$int_0^1 frac{1}{x^m+x^{m+1}}dx$$



                    and since as $xto 0^+$



                    $$frac{1}{x^m+x^{m+1}} sim frac{1}{x^m}$$



                    the integral converges for $m<1$ that is $n>-1$.



                    As an alternative by $y=frac1x$ we have



                    $$int_0^1 frac{x^n}{1+x}dx=int_1^infty frac{1}{y^{n+1}+y^{n+2}}dx$$



                    and since as $xto infty$



                    $$ frac{1}{y^{n+1}+y^{n+2}}sim frac{1}{y^{n+2}}$$



                    the integral converges for $n+2>1$ that is $n>-1$.






                    share|cite|improve this answer












                    We have that for $nge 0$ the integral is a proper integral, then consider $n<0$ and by $m=-n>0$ we have



                    $$int_0^1 frac{1}{x^m+x^{m+1}}dx$$



                    and since as $xto 0^+$



                    $$frac{1}{x^m+x^{m+1}} sim frac{1}{x^m}$$



                    the integral converges for $m<1$ that is $n>-1$.



                    As an alternative by $y=frac1x$ we have



                    $$int_0^1 frac{x^n}{1+x}dx=int_1^infty frac{1}{y^{n+1}+y^{n+2}}dx$$



                    and since as $xto infty$



                    $$ frac{1}{y^{n+1}+y^{n+2}}sim frac{1}{y^{n+2}}$$



                    the integral converges for $n+2>1$ that is $n>-1$.







                    share|cite|improve this answer












                    share|cite|improve this answer



                    share|cite|improve this answer










                    answered Nov 24 at 18:05









                    gimusi

                    1




                    1























                        0














                        Let $a_n = displaystyle int_{0}^1 dfrac{x^n}{1+x}dx$. Since $dfrac{x^n}{1+x} = x^{n-1}left(1-dfrac{1}{1+x}right)=x^{n-1}-dfrac{x^{n-1}}{1+x}$, taking the integral $displaystyle int_{0}^1$ both sides give: $a_n = dfrac{1}{n}- a_{n-1}=dfrac{1}{n}-dfrac{1}{n-1}+a_{n-2}= dfrac{1}{n}-dfrac{1}{n-1}+dfrac{1}{n-2}-a_{n-3}=...=-ln 2+ 1-dfrac{1}{2}+dfrac{1}{3}-dfrac{1}{4}+dfrac{1}{5}-dfrac{1}{6}+cdots + dfrac{1}{n-2}-dfrac{1}{n-1}+dfrac{1}{n}$. $a_n$ converges when considered as a partial sum of an alternating harmonic series.






                        share|cite|improve this answer




























                          0














                          Let $a_n = displaystyle int_{0}^1 dfrac{x^n}{1+x}dx$. Since $dfrac{x^n}{1+x} = x^{n-1}left(1-dfrac{1}{1+x}right)=x^{n-1}-dfrac{x^{n-1}}{1+x}$, taking the integral $displaystyle int_{0}^1$ both sides give: $a_n = dfrac{1}{n}- a_{n-1}=dfrac{1}{n}-dfrac{1}{n-1}+a_{n-2}= dfrac{1}{n}-dfrac{1}{n-1}+dfrac{1}{n-2}-a_{n-3}=...=-ln 2+ 1-dfrac{1}{2}+dfrac{1}{3}-dfrac{1}{4}+dfrac{1}{5}-dfrac{1}{6}+cdots + dfrac{1}{n-2}-dfrac{1}{n-1}+dfrac{1}{n}$. $a_n$ converges when considered as a partial sum of an alternating harmonic series.






                          share|cite|improve this answer


























                            0












                            0








                            0






                            Let $a_n = displaystyle int_{0}^1 dfrac{x^n}{1+x}dx$. Since $dfrac{x^n}{1+x} = x^{n-1}left(1-dfrac{1}{1+x}right)=x^{n-1}-dfrac{x^{n-1}}{1+x}$, taking the integral $displaystyle int_{0}^1$ both sides give: $a_n = dfrac{1}{n}- a_{n-1}=dfrac{1}{n}-dfrac{1}{n-1}+a_{n-2}= dfrac{1}{n}-dfrac{1}{n-1}+dfrac{1}{n-2}-a_{n-3}=...=-ln 2+ 1-dfrac{1}{2}+dfrac{1}{3}-dfrac{1}{4}+dfrac{1}{5}-dfrac{1}{6}+cdots + dfrac{1}{n-2}-dfrac{1}{n-1}+dfrac{1}{n}$. $a_n$ converges when considered as a partial sum of an alternating harmonic series.






                            share|cite|improve this answer














                            Let $a_n = displaystyle int_{0}^1 dfrac{x^n}{1+x}dx$. Since $dfrac{x^n}{1+x} = x^{n-1}left(1-dfrac{1}{1+x}right)=x^{n-1}-dfrac{x^{n-1}}{1+x}$, taking the integral $displaystyle int_{0}^1$ both sides give: $a_n = dfrac{1}{n}- a_{n-1}=dfrac{1}{n}-dfrac{1}{n-1}+a_{n-2}= dfrac{1}{n}-dfrac{1}{n-1}+dfrac{1}{n-2}-a_{n-3}=...=-ln 2+ 1-dfrac{1}{2}+dfrac{1}{3}-dfrac{1}{4}+dfrac{1}{5}-dfrac{1}{6}+cdots + dfrac{1}{n-2}-dfrac{1}{n-1}+dfrac{1}{n}$. $a_n$ converges when considered as a partial sum of an alternating harmonic series.







                            share|cite|improve this answer














                            share|cite|improve this answer



                            share|cite|improve this answer








                            edited Nov 24 at 18:46

























                            answered Nov 24 at 18:24









                            DeepSea

                            70.9k54487




                            70.9k54487























                                0














                                Since $x^n$ is continuous on $[0,1]$ for $nge 0,$ the integral converges for $nge 0.$



                                For $n<0,$ $x^n$ blows up at $0.$ So we need to consider the integral over $[a,1]$ for small $a>0.$ Notice that for $xin (0,1],$



                                $$frac{x^n}{2}le frac{x^n}{1+x} le x^n.$$



                                It follows that the integral of interest converges iff $int_0^1 x^n,dx $ converges. Things are easy now, so I'll stop here. Ask questions if you like.






                                share|cite|improve this answer


























                                  0














                                  Since $x^n$ is continuous on $[0,1]$ for $nge 0,$ the integral converges for $nge 0.$



                                  For $n<0,$ $x^n$ blows up at $0.$ So we need to consider the integral over $[a,1]$ for small $a>0.$ Notice that for $xin (0,1],$



                                  $$frac{x^n}{2}le frac{x^n}{1+x} le x^n.$$



                                  It follows that the integral of interest converges iff $int_0^1 x^n,dx $ converges. Things are easy now, so I'll stop here. Ask questions if you like.






                                  share|cite|improve this answer
























                                    0












                                    0








                                    0






                                    Since $x^n$ is continuous on $[0,1]$ for $nge 0,$ the integral converges for $nge 0.$



                                    For $n<0,$ $x^n$ blows up at $0.$ So we need to consider the integral over $[a,1]$ for small $a>0.$ Notice that for $xin (0,1],$



                                    $$frac{x^n}{2}le frac{x^n}{1+x} le x^n.$$



                                    It follows that the integral of interest converges iff $int_0^1 x^n,dx $ converges. Things are easy now, so I'll stop here. Ask questions if you like.






                                    share|cite|improve this answer












                                    Since $x^n$ is continuous on $[0,1]$ for $nge 0,$ the integral converges for $nge 0.$



                                    For $n<0,$ $x^n$ blows up at $0.$ So we need to consider the integral over $[a,1]$ for small $a>0.$ Notice that for $xin (0,1],$



                                    $$frac{x^n}{2}le frac{x^n}{1+x} le x^n.$$



                                    It follows that the integral of interest converges iff $int_0^1 x^n,dx $ converges. Things are easy now, so I'll stop here. Ask questions if you like.







                                    share|cite|improve this answer












                                    share|cite|improve this answer



                                    share|cite|improve this answer










                                    answered Nov 24 at 19:04









                                    zhw.

                                    71.6k43075




                                    71.6k43075






























                                        draft saved

                                        draft discarded




















































                                        Thanks for contributing an answer to Mathematics Stack Exchange!


                                        • Please be sure to answer the question. Provide details and share your research!

                                        But avoid



                                        • Asking for help, clarification, or responding to other answers.

                                        • Making statements based on opinion; back them up with references or personal experience.


                                        Use MathJax to format equations. MathJax reference.


                                        To learn more, see our tips on writing great answers.





                                        Some of your past answers have not been well-received, and you're in danger of being blocked from answering.


                                        Please pay close attention to the following guidance:


                                        • Please be sure to answer the question. Provide details and share your research!

                                        But avoid



                                        • Asking for help, clarification, or responding to other answers.

                                        • Making statements based on opinion; back them up with references or personal experience.


                                        To learn more, see our tips on writing great answers.




                                        draft saved


                                        draft discarded














                                        StackExchange.ready(
                                        function () {
                                        StackExchange.openid.initPostLogin('.new-post-login', 'https%3a%2f%2fmath.stackexchange.com%2fquestions%2f3011862%2ffinding-convergence-of-integral-int-01-fracxn1xdx%23new-answer', 'question_page');
                                        }
                                        );

                                        Post as a guest















                                        Required, but never shown





















































                                        Required, but never shown














                                        Required, but never shown












                                        Required, but never shown







                                        Required, but never shown

































                                        Required, but never shown














                                        Required, but never shown












                                        Required, but never shown







                                        Required, but never shown







                                        Popular posts from this blog

                                        Plaza Victoria

                                        In PowerPoint, is there a keyboard shortcut for bulleted / numbered list?

                                        How to put 3 figures in Latex with 2 figures side by side and 1 below these side by side images but in...